Fernando Garcia

Home     Research     Blog     Other     About me


PHYS405 - Electricity & Magnetism I

Spring 2024

Problem 1.63

(a) Find the divergence of the function

\[\mathbf{v}=\frac{1}{r}\hat{\mathbf{r}} \]

First compute it directly, as in Eq. 1.84:

\[\nabla \cdot \left( \frac{1}{r^2 }\hat{\mathbf{r}} \right) =\frac{1}{r^2 }\frac{\partial }{\partial r} \left( r^2 \frac{1}{r^2 } \right) =\frac{1}{r^2 }\frac{\partial }{\partial r}(1)=0\tag{Eq. 1.84, page 43} \]

Test your result using the divergence theorem (over a sphere of radius $R $, centered at the origin), as in Eq. 1.85:

\[\oint \mathbf{v}\cdot d\mathbf{a}=\int \left( \frac{1}{R^2 }\hat{\mathbf{r}} \right) \cdot (R^2 \sin \theta d\theta d\phi \hat{\mathbf{r}})\tag{Eq. 1.85, page 44} \]

Is there a delta function at the origin, as there was for $\hat{\mathbf{r}}/r^2 ? $ What is the general formula for the divergence of $r^n \hat{\mathbf{r}} $?

(b) Find the curl of $r^n \hat{\mathbf{r}} $. Test your conclusion using problem 1.61b.

(a) A direct computation gives

\begin{align*} \nabla \cdot \mathbf{v} &= \nabla \cdot \left( \frac{1}{r}\hat{\mathbf{r}} \right) \\ &= \frac{1}{r^2 }\frac{\partial }{\partial r} \left( r^2 \frac{1}{r} \right) \\ &= \frac{1}{r^2 }\frac{\partial }{\partial r}(r)\\ &= \frac{1}{r^2 } \end{align*}

Using the divergence theorem

\[\int \mathbf{v}\cdot d\mathbf{a}=\int \nabla \cdot \mathbf{v}d\tau \]

let's first take a surface integral over a sphere of radius $R $:

\begin{align*} \oint \mathbf{v}\cdot d\mathbf{a} &= \int \left( \frac{1}{R}\hat{\mathbf{r}} \right) \cdot (R^2 \sin \theta d\theta d\phi \hat{\mathbf{r}} ) \\ &= R\int_{\phi =0}^{2\pi } \int_{\theta =0}^{\pi } \sin \left( \theta \right) d\theta d\phi \\ &= 4\pi R \end{align*}

And now a volume integral of the divergence

\begin{align*} \int \nabla \cdot \mathbf{v}d\tau &= \int_{\phi =0}^{2\pi } \int_{\theta =0}^{\pi } \int_{r=0}^{R} \left( \frac{1}{r^2 } \right) \left( r^2 \sin \left( \theta \right) drd\theta d\phi \right) \\ &= 4\pi R \end{align*}

In this case (contrary to $\mathbf{v}=\hat{\mathbf{r}}/r^2 $), the divergence theorem works perfectly.

In general, we have:

\begin{align*} \nabla \cdot (r^n \hat{\mathbf{r}}) &= \frac{1}{r^2 }\frac{\partial }{\partial r} \left( r^2 r^n \right) \\ &= \frac{1}{r^2 }\frac{\partial }{\partial r}(r^{n+2}) \\ &= \frac{1}{r^2 }(n+2)r^{n+1} \\ &= (n+2)r^{n+1-2} \\ &= (n+2)r^{n-1} \tag{For $n\neq -2 $} \end{align*}

If $n=-2 $, then we have the delta Dirac $\nabla \cdot (\hat{\mathbf{r}}/r^2 )=4\pi \delta ^3 (\mathbf{r}) $ (see Eq. 1.99, page 48).

(b) A quick inspection at the curl formula in spherical coordinates tells us that

\[\nabla \times (r^n \hat{\mathbf{r}})=\mathbf{0} \]

Using problem 1.61b, we see that the integrand on right hand side of

\[\int _{\mathcal{V}}(\nabla \times \mathbf{v})d\tau =-\oint _{\mathcal{S}}\mathbf{v}\times d\mathbf{a} \]

Is a cross product of parallel vectors and thus vanishes. This means that the left hand side is also zero, as expected from the explicit computation of the curl.


Back to PHYS405 site